LSAT and Law School Admissions Forum

Get expert LSAT preparation and law school admissions advice from PowerScore Test Preparation.

 Administrator
PowerScore Staff
  • PowerScore Staff
  • Posts: 8917
  • Joined: Feb 02, 2011
|
#28488
Complete Question Explanation
(The complete setup for this game can be found here: lsat/viewtopic.php?t=9894)

The correct answer choice is (A)

From our analysis of the two sequence templates, we know that in template #1 only H and K can be the most expensive. This information eliminates answer choices (C) and (E). In template #2 only G and J can be the most expensive, and that eliminates answer choice (B) and (D). Thus, answer choice (A) is proven correct by process of elimination.

Alternatively, answer choice (A) can be proven correct because in template #1, F must be less expensive than H, and in template #2, F must be less expensive than G.

Answer choice (A): This is the correct answer choice.

Answer choices (B) and (D): These two answer choices are incorrect because sequence template #2 allows for G or J to be the most expensive suite.

Answer choices (C) and (E): These two answer choices are incorrect because sequence template #1 allows for H or K to be the most expensive suite.

Get the most out of your LSAT Prep Plus subscription.

Analyze and track your performance with our Testing and Analytics Package.